Akademisyenler öncülüğünde matematik/fizik/bilgisayar bilimleri soru cevap platformu
3 beğenilme 0 beğenilmeme
558 kez görüntülendi
Serbest kategorisinde (1.8k puan) tarafından 
tarafından düzenlendi | 558 kez görüntülendi

$a_2.a_3 \dots, a_n=1$  de bir hata var mı?

Yok sayın Hocam, ben ilk yazarken unutmuşum siz düzenlemeyi yapınca farkettim, ben sonradan ekledim. bir defa daha kontol ettim şimdi yukarıdaki gibi orjinal metinde

yani carpimi 1 mi ? oradaki virgul kafa karistiriyor.

evet hocam çarpımı bir o benim yazamamamdan kaynaklanır virgül olmayacak

Çarpımı açtığımızda  $1+n!+$$\displaystyle{2\choose 2}  {3\choose 2}...{n\choose 2} + \mathbb{bazı terimler}$ gibi şeyler buldum. Buradan sezgisel olarak $\displaystyle\frac{(n^n)!}{n!}$ gibi bir şey çıkacağını düşünüyorum ama daha bir sonuca ulaşamadım.

1 cevap

2 beğenilme 0 beğenilmeme
En İyi Cevap

2012 Shortlist sorularının hepsinin cevabı biliniyor. Gene de  buraya cevabı ekliyorum.  Çözüm Aritmetik-geometrik Ortalama eşitsizliğine ve $1$ sayısını $\frac{1}{k}$ sayısının $k$ tanesinin toplamı olarak yazma fikrine dayanıyor.

$(\frac{1+a_2}{2})^2 \ge a_2$

$(\frac{\frac{1}{2}+\frac{1}{2}+a_3}{3})^3 \ge \frac{1}{2^2}a_3$

$(\frac{\frac{1}{3}+\frac{1}{3}+\frac{1}{3}+a_4}{4})^4 \ge \frac{1}{3^3}a_4$

$\cdots$

$(\frac{\frac{1}{n-1}+\frac{1}{n-1}+\cdots + \frac{1}{n-1}+a_n}{n})^n \ge \frac{1}{(n-1)^{n-1}}a_n$

Bu eşitsizlikler taraf, tarafa çarpılır, gerekli kısaltmalar yapılır ve $a_2a_3 \cdots a_n=1$ olduğu göz önüne alınırsa

$(1+a_2)^2 (1+a_3)^3\cdots (1+a_n)^n \ge n^n$

bulunur. Eşitliğin olması için $a_2=1$, $a_3=\frac{1}{2}$, $\cdots$, $a_n=\frac{1}{2}$ olması gerekir. Fakat bu durumda $a_2a_3 \cdots a_n=1$ olamaz. O halde eşitsizlik kesindir.  

(541 puan) tarafından 
tarafından seçilmiş

Çok teşekkürler hocam. güzel bir çözüm

20,200 soru
21,728 cevap
73,275 yorum
1,887,895 kullanıcı